Wall Quiz #6 Flashcards

1
Q

How do you counsel regarding the mechanism of Levonorgesterol (LNG) IUD and how it prevents pregnancy?
A. Anovulation
B. Increasing viscosity of mucous of the cervix
C. Sterile inflammation of uterine wall
D. Atrophy of endometrium
E. Inhibition of FSH

A

B. Increasing viscosity of mucous of the cervix

How well did you know this?
1
Not at all
2
3
4
5
Perfectly
2
Q
A chorioangioma greater than 5cm can be associated with all of the following EXCEPT:
A. Normal Pregnancy
B. Arterial-venous shunting 
C. Fetal anemia
D. Thrombosis
E. Fetal growth restriction
A

D. Thrombosis

How well did you know this?
1
Not at all
2
3
4
5
Perfectly
3
Q
Which of the following hysterectomy surgical approaches would be most appropriate for an 39 year old with a 7 week symptomatic fibroid uterus and a BMI of 42?
A. Midline incision
B. Low transverse incision 
C. Laparoscopic
D. Incision at palmers point 
E. Vaginal approach
A

E. Vaginal approach

How well did you know this?
1
Not at all
2
3
4
5
Perfectly
4
Q
A 17 y/o G0 requests updating all her vaccinations prior to leaving for college. All of the following immunizations should be updated EXCEPT:
A. Tdap
B. Meningococcal 
C. Influenza
D. MMR
E. Pneumococcal
A

E. Pneumococcal

How well did you know this?
1
Not at all
2
3
4
5
Perfectly
5
Q
5. After a maternal trauma if one of the following completed, which radiological examination had the lowest fetal dose during exposure?
A. Chest CT
B. Abdominal radiography 
C. Head or neck CT
D. Chest radiography
E. Mammography
A

D. Chest radiography

How well did you know this?
1
Not at all
2
3
4
5
Perfectly
6
Q

All of the following are benefits of delayed umbilical cord clamping EXCEPT:
A. Improvement in blood count
B. Decreased need for transfusion
C. Decreased incidence of sepsis
D. Lower rates of Intraventricular hemorrhage
E. Improved iron stores

A

C. Decreased incidence of sepsis

How well did you know this?
1
Not at all
2
3
4
5
Perfectly
7
Q
A 16-year-old transgender male presents for cross puberty induction. What is the recommended treatment of choice?
A. GnRH agonist
B. Norethindrone
C. Transdermal estrogen 
D. Testosterone
E. Spironolactone
A

D. Testosterone

How well did you know this?
1
Not at all
2
3
4
5
Perfectly
8
Q
Where is the highest concentration of C02 in the fetal heart in utero?
A. Left ventricle 
B. Left atrium
C. Right atrium
D. Right ventricle 
E. Umbilical artery
A

D. Right ventricle

How well did you know this?
1
Not at all
2
3
4
5
Perfectly
9
Q
Thyroid stimulating hormone may be decreased in the first trimester due to an increase in?
A. Prematurity
B. Thyroid binding globulin
C. Human chorionic gonadotropin 
D. Progesterone
E. Estrogen
A

C. Human chorionic gonadotropin

How well did you know this?
1
Not at all
2
3
4
5
Perfectly
10
Q
Which of the following bacteria are least likely causative in chorioamnionitis?
A. Ureaplasma 
B. Bacteroides 
C. Gardnerella 
D. Listeria
E. Mycoplasma
A

D. Listeria

How well did you know this?
1
Not at all
2
3
4
5
Perfectly
11
Q
What is the correct ratio of chest compressions to breaths per minute in the resuscitation of a term neonate?
A. 30 compressions: 90 breaths 
B. 60 compressions: 15 breaths 
C. 90 compressions: 30 breaths 
D. 60 compressions: 30 breaths 
E. 40 compressions: 15 breaths
A

C. 90 compressions: 30 breaths

How well did you know this?
1
Not at all
2
3
4
5
Perfectly
12
Q
A 21 y/o G1P0 presents for her first annual visit. Her Hgb is 9.8g/dL and the MCV is 84. What is the diagnosis if the reticulocyte count is low?
A. Megaloblastic anemia
B. Thalassemia
C. Non megaloblastic anemia
D. Sickle cell anemia
E. Combined iron and folate deficiency
A

E. Combined iron and folate deficiency

How well did you know this?
1
Not at all
2
3
4
5
Perfectly
13
Q

When is Herceptin (Trastuzumab) best used in the adjuvant treatment of breast cancer?
A. Estrogen receptor negative breast cancer
B. Positive HER2/neu receptor breast cancer
C. Estrogen receptor positive breast cancer
D. Progesterone receptor positive breast cancer
E. Negative HER2/neu receptive breast cancer

A

B. Positive HER2/neu receptor breast cancer

How well did you know this?
1
Not at all
2
3
4
5
Perfectly
14
Q

Which of the following factors is best in determining the prognosis of breast cancer?
A. Size of tumor
B. Number of lymph nodes
C. Presence of estrogen and progesterone receptors
D. Histological grade
E. Stage of disease

A

E. Stage of disease

How well did you know this?
1
Not at all
2
3
4
5
Perfectly
15
Q
Which of the following tumor markers is most reliable during pregnancy?
A. CA 125 
B. CEA 
C. LDH 
D. AFP
E. hCG
A

C. LDH

How well did you know this?
1
Not at all
2
3
4
5
Perfectly
16
Q
A 19-year-old is referred to you from family practice due to an elevated AM prolactin of 101 ng/mL and irregular menses. What is the first step in your evaluation?
A. Prescribe cabergoline 
B. Repeat Prolactin level 
C. MRI of the Brain
D. Oral contraceptives 
E. CT of the Brain
A

C. MRI of the Brain

How well did you know this?
1
Not at all
2
3
4
5
Perfectly
17
Q

A 67-year-old patient with metastatic ovarian cancer presents with abdominal pain. On evaluation, you are suspicious of an increased intra-abdominal pressure. What is the next step in evaluation?
A. Sedation and analgesia
B. Nasogastric tube and intermittent suction
C. Surgical decompression
D. Monitor intraabdominal pressures
E. Paracentesis

A

D. Monitor intraabdominal pressures

How well did you know this?
1
Not at all
2
3
4
5
Perfectly
18
Q
Which hormone increases/peaks only in the mid- luteal phase during a normal menstrual cycle?
A. Estrogen
B. Luteinizing hormone (LH) 
C. Inhibin A
D. Inhibin B
E. FSH
A

C. Inhibin A

How well did you know this?
1
Not at all
2
3
4
5
Perfectly
19
Q
Which of the following is the most likely predisposing risk factor in a 75-year-old woman diagnosed with vulvar cancer?
A. Cigarette smoking
B. Obesity
C. Age
D. Multiple High-Risk HPV Paps 
E. Race
A

C. Age

How well did you know this?
1
Not at all
2
3
4
5
Perfectly
20
Q

Which of the following is the best treatment option for a 45-year-old nulliparous woman with stage IBI squamous cell carcinoma of the cervix?
A. Primary radiation and chemotherapy
B. Radical hysterectomy and pelvic lymphadenectomy
C. Radical hysterectomy and chemotherapy
D. Primary radiation therapy
E. Simple hysterectomy

A

B. Radical hysterectomy and pelvic lymphadenectomy

How well did you know this?
1
Not at all
2
3
4
5
Perfectly
21
Q
Which of the following is associated with HLA- DR4?
A. Rheumatoid arthritis
B. Systematic Lupus Erythmatoses 
C. Crohns disease
D. Inflammatory bowel disease
E. Osteoarthritis
A

A. Rheumatoid arthritis

22
Q

Which scenario most warrants BRCA testing?
A. Breast CA at age 55; No family history of Breast CA
B. No Breast/Ovarian CA; Aunt with Breast CA at age 59
C. Breast CA at age 49; Unknown family history
D. No Breast CA; Sibling with Ovarian CA at age 60
E. History of Ovarian Granulosa Cell CA at age 55

A

C. Breast CA at age 49; Unknown family history

23
Q

A 25-year-old G2P1 at 18 weeks has a negative Zika virus NAT and non-negative Zika virus IgM. Which of the following is most useful for establishing the diagnosis of Zika?
A. Chorionic villus sampling and repeat NAT testing
B. Ultrasound of fetal cranium
C. Avidity testing
D. Plaque reduction neutralization test
E. Repeat Zika Virus IgM test

A

D. Plaque reduction neutralization test

24
Q
Which of the following cardio-pulmonary disorders has the worst maternal prognosis?
A. Aortic stenosis
B. Mitral regurgitation
C. Aortic regurgitation
D. Pulmonary hypertension 
E. Patent ductus arteriosus
A

D. Pulmonary hypertension

25
Q
Which of the following ligaments provides the major support to the uterus?
A. Round
B. Cardinal
C. Umbilical
D. Uterosacral
E. Infundibulopelvic
A

B. Cardinal

26
Q
The colposcopic finding most indicative of invasive cervical carcinoma is?
A. Mosaicism
B. Punctation
C. Acetowhitening 
D. Atypical vessels 
E. Hyperkeratosis
A

D. Atypical vessels

27
Q
For a 70-year-old patient presenting for her yearly evaluation, which of the following should be recommended on an annual basis?
A. Bone density
B. Colonoscopy
C. Pneumococcal vaccination 
D. Influenza vaccination
E. TSH
A

D. Influenza vaccination

28
Q
Which of the following tests must be performed before (Rho[D])immune globulin is administered to an unsensitized D-negative primagravida patient?
A. Indirect Coombs test of the neonate 
B. Indirect Coombs test of the mother 
C. Direct Coombs test of the mother 
D. Direct Coombs test of the neonate 
E. ABO typing of neonate
A

B. Indirect Coombs test of the mother

29
Q

The treatment of choice for stage II invasive carcinoma of the vagina is:
A. Radiation therapy
B. Laser vaporization
C. Radical vaginectomy
D. 5-Fluorouracil
E. Administration of cisplatin followed by wide local incision

A

A. Radiation therapy

30
Q

Rho[D] immune globulin most likely prevents D isoimmunization by:
A. Peripheral lysis of erythrocytes
B. Blocking D antigenic determinants on the erythrocyte membrane
C. Preventing D Antigen from reaching the germinal centers of lymph nodes
D. Forming immune globulin immune complexes

A

B. Blocking D antigenic determinants on the erythrocyte membrane

31
Q
The most common presentation of a patient with erosive lichen planus of the vagina is:
A. Dyspareunia
B. Pruritus
C. No symptoms
D. Post coital bleeding 
E. Watery discharge
A

A. Dyspareunia

32
Q
Pseudomyxoma peritonei can be fatal if left untreated due to which of the following?
A. Bowel obstruction
B. Vascular seeding and occlusion 
C. Ureteral obstruction
D. Chemical peritonitis
E. Ascites
A

A. Bowel obstruction

33
Q
A 25-year-old G1 at 12 weeks gestation presents with pediculosis pubis. Of the following medications, the most appropriate to use is?
A. 2% ketoconazole cream 
B. 1% hydrocortisone cream 
C. Lindane
D. Benzoyl peroxide
E. Permethrin 5% cream
A

E. Permethrin 5% cream

34
Q
Which of the following is the leading cause of septic shock in pregnancy?
A. IV Drug Abuse
B. Pelvic Inflammatory Disease 
C. Untreated Gonorrheal Infection 
D. Pyelonephritis
E. Chorioamnionitis
A

D. Pyelonephritis

35
Q
A 25 year old presents with an asymptomatic 4 cm simple ovarian cyst of the right ovary. Which of the following is the next best step?
A. Repeat sonographic imaging 
B. Tumor markers
C. Laparoscopy
D. CT guided drainage
E. Laparoscopic guided needle aspiration
A

A. Repeat sonographic imaging

36
Q
Which of the following is a risk factor for shoulder dystocia?
A. Maternal gestational diabetes 
B. Fetal hydrocephalus
C. Fetal prematurity
D. Prolonged Oxytocin use
E. Maternal BMI 25
A

A. Maternal gestational diabetes

37
Q
Which of the following is the most significant risk factor for the development of an ectopic pregnancy?
A. Prior chlamydial infection 
B. History of tubal ligation 
C. Prior molar pregnancy
D. Prior miscarriage
E. Combination oral contraceptive use
A

A. Prior chlamydial infection

38
Q
What is the next step in the management of a 9- year-old with a unilateral mass posterior to the areola:
A. Mammography 
B. U/S
C. Biopsy
D. Observation
E. MRI
A

D. Observation

39
Q
The perinatal period is defined as the interval from 20 weeks gestation to:
A. 7 days postpartum
B. 28 days postpartum
C. 1 calendar month after delivery 
D. 6 weeks postpartum
E. 1 year after delivery
A

B. 28 days postpartum

40
Q

When applying the suction cup for a vacuum delivery, where should the placement be(flexion point)?
A. 2cm posterior to the anterior fontanelle
B. Midline to the coronal suture
C. Anterior on the sagittal suture, 3cm
D. Midline to the sagittal suture, 2cm
E. 3cm anterior to the posterior fontanelle

A

E. 3cm anterior to the posterior fontanelle

41
Q
A 44 y/o presents with a history of Peripheral artery disease, and Hypertension with an LDL – 180. What should her goal LDL be?
A. Less than 100 mg/dL 
B. Less than 130 mg/dL 
C. Less than 160 mg/dL 
D. Less than 190 mg/dL 
E. Less than 200 mg/dL
A

A. Less than 100 mg/dL

42
Q
Fecundity declines gradually but significantly beginning at approximately what age?
A. 32 
B. 33 
C. 34 
D. 35 
E. 36
A

A. 32

43
Q
Which of the following percentage is the upper limit of discordance recommended to safely extract a second twin breech, assuming the presenting twin is smaller?
A. 1% 
B. 5% 
C. 10% 
D. 15% 
E. 20%
A

E. 20%

44
Q
A patient presents with necrotizing fasciitis; cultures show: Beta-hemolytic/Catalase Negative/gram-positive cocci.
What is the most likely organism?
A. Staphylococcus aureus
B. E. coli
C. Staphylococcus epidermidis
D. Streptococcus pyogenes
E. Streptococcaceae pneumoniae
A

D. Streptococcus pyogenes

45
Q
Which of the following is NOT a characteristic of monochorionic placentation by ultrasound?
A. T sign
B. Less than 2mm membrane thickness 
C. Gender concordant
D. 3 visible membrane layers
E. One placenta
A

D. 3 visible membrane layers

46
Q
When counseling a patient on the HPV vaccination; Which of the following allergies would be of concern?
A. Albumin
B. Aluminum 
C. Eggs
D. Thimerosal 
E. Yeast
A

E. Yeast

47
Q
A White/Caucasian couple presents for pre- conceptional counseling regarding cystic fibrosis. Both are positive carriers for the cystic fibrosis gene. What is the likelihood they will have a male fetus with the disease?
A. 25% 
B. 12.5% 
C. 75% 
D. 100% 
E. 5%
A

B. 12.5%

48
Q
A 27-year-old G0 presents with bilateral hydrosalpinx and infertility. What is the next best step to increase her chances of fertility with IVF?
A. Antibiotics
B. HSG
C. Bilateral salpingostomy 
D. Bilateral salpingectomy 
E. Chromopertubation
A

D. Bilateral salpingectomy

49
Q

Which of the following describes level one support of the vagina and uterus?
A. Arcus tendineus fasciae pelvis
B. Perineal body
C. Uterosacral/cardinal ligament complex
D. Superficial transverse perineus muscle
E. External anal sphincter

A

C. Uterosacral/cardinal ligament complex

50
Q
Which of the following is not a recommended treatment for EIN – Endometrial Intraepithelial Neoplasia?
A. Megace
B. Micronized progesterone 
C. Hysterectomy
D. DMPA
E. Etonogesterol implant
A

E. Etonogesterol implant